GMAT 考满分题库

GWD - 逻辑CR - 200
Which of the following, if true, provides evidence that most logically completes the argument below?

According to a widely held economic hypothesis, imposing strict environmental regulations reduces economic growth. This hypothesis is undermined by the fact that the states with the strictest environmental regulations also have the highest economic growth. This fact does not show that environmental regulations promote growth, however, since ______.
  • Athose states with the strictest environmental regulations invest the most in education and job training 分析该选项
  • Beven those states that have only moderately strict environmental regulations have higher growth than those with the least-strict regulations 分析该选项
  • Cmany states that are experiencing reduced economic growth are considering weakening their environmental regulations 分析该选项
  • Dafter introducing stricter environmental regulations, many states experienced increased economic growth 分析该选项
  • Eeven those states with very weak environmental regulations have experienced at least some growth 分析该选项
显示答案
正确答案: A

讨论题目 或 发起提问

|

题目讨论

  • 按热度
  • 按顺序

最新提问